PT50.S2.Q16 - multiparty democracies

babybennybabybenny Free Trial Member
edited December 2015 in Logical Reasoning 156 karma
http://7sage.com/lsat_explanations/lsat-50-section-2-question-16/
I have two questions about this question.

1) Which sentence is the conclusion in the stimulus?
I'm confused between the first and last sentence.

2) I was down to B and C, and am having a difficult time understanding why B is incorrect.
Can someone please explain me?

Thanks in advance!
Sign In or Register to comment.